1. The diagram shows a semicircle OABC. If the arc AB has length 3.2 cm, calculate (a) the length of the radius, (b) the length of the arc BC. r 0.8 rad С o r A​

Answers

Answer 1

Step-by-step explanation:

— If the arc AB has length 3.2 cm, calculate (a) the length of the radius, (b) ... the length of the radius, (b) the length of the arc BC. r 0.8 rad С o r A .


Related Questions

If x < 0 and y < 0, what guadrant is the point (x, y) located

Answers

Answer: Third quadrant

This can be written as Q3 or QIII

This quadrant is in the southwest.

===========================================

Explanation:

There are 4 quadrants on the xy plane. They are separated by the x and y axis. Think of it like 4 separate rooms.

The first quadrant Q1 is located in the northeast where x > 0 and y > 0. Both x and y are positive together. An example point from this region would be something like (5,2)The second quadrant, Q2 or QII, is in the northwest. An example point from here is (-5,2). We can see that x < 0 and y > 0.The third quadrant (Q3) is where both x and y are negative. So x < 0 and y < 0. An example point would be (-5,-2). This quadrant is in the southwest.Finally, if x > 0 and y < 0, then we're in the southeast quadrant Q4. An example point is (5,-2)

As you can see, the quadrants move in a counterclockwise fashion when going from Q1 to Q2 to Q3 to Q4.

Quadrants 1 and 4 have x,y that are the same sign together. Either they're both positive together, or they're both negative together. Quadrants 2 and 3 have x,y as opposite signs (one is positive and the other is negative).

If Clare earns $75 the next week from delivering newspapers and deposits it in her account, What will her account balance be then?\


answer pls

Answers

Answer: $15

Step-by-step explanation:

-$50 + $75 = $15

Does (-54, -90) make the equation y = x - -36 true? yes no​

Answers

Not this is not true, -54 - -36 equals -18

if the parent function is y=3^x, which is the function of the graph?
1) y=3^(x-4)-2
2) y=3^(x+4)+2
3) y=3^(x-4)+2
4) y=3^(x+4)-2

Answers

3 because it equal yhhh

Someone tell me where everyone is going right please !!

Answers

Answer:

1. 3rd option

2. 4th option

3. 1st option

4. 2nd option

5. 2nd option

Step-by-step explanation:

1.

on changing the signs of an equality the sign gets reveresd

-6 < -2x

6 > 2x

3 > x

2.

2x - 3 > 11 - 5x

adding 5x and 3 to both sides and combining like terms

(2x + 5x) + (-3 + 3) > (11 + 3) + (-5x + 5x)

7x > 14

x > 2

put of all the options x = 4 is a value greater than 2 thus satisfying the condition.

3.

6k + 10. 5 = 3k + 12

subtracting 3k and 10.5 from both the sides while combining like terms

(6k - 3k) + (10.5 - 10.5) = (3k - 3k) + (12 - 10.5)

3k = 1.5

k = 0.5

4.

y + 3 = -y + 9

adding y and subtracting 3 from both sides

(y + y) + (3 - 3) = (-y + y) + (9 - 3)

2y = 6

y = 3

5.

-9x + 1 = -x + 17

adding x and subtracting 1 from both sides

(-9x + x) + (1 -1) = (-x +x) + (17 -1)

-8x = 16

multiplying the equation by -1

8x = -16

diving both sides by 8

x = -2

Answer:

[tex]1. \: \: x < 3[/tex]

[tex]2. \: \: 2[/tex]

[tex]3. \: \: k = 0.5[/tex]

[tex]4. \: \: y = 3[/tex]

[tex]5. \: \: x = 12[/tex]

Step-by-step explanation:

[tex]1. \: \: - 2(5 - 4x) < 6x - 4 \\ - 10 + 8x < 6x - 4 \\ - 10 + 2x < - 4 \\ 2x < 6 \\ x < 3[/tex]

[tex]2. \: \: 2x - 3 > 11 - 5x \\ 7x - 3 > 11 \\ 7x > 14 \\ x = 2[/tex]

[tex]3. \: \: 6k + 10.5 = 3k + 12 \\ 3k + 10.5 = 12 \\ 3k = 1.5 \\ k = 0.5[/tex]

[tex]4. \: \: y + 3 = - y + 9 \\ 2y + 3 = 9 \\ 2y = 6 \\ y = 3[/tex]

[tex]5. \: \: - 9x + 1 = - x + 17 \\ - 8x + 1 = 17 \\ - 8x = 16 \\ x = - 2[/tex]

Hope it is helpful....

Which number line represents the solutions to -2x = -6?

Answers

Answer:

3

Step-by-step explanation:

-2x = -6

or, x = -6 / -2

or, x = 3

Can someone help me please this is my last one ​

Answers

Answer:

Domain and range: [0, ∞)

Step-by-step explanation:

There is no limit to the number of concert tickets someone can buy, but there cannot be negative concert tickets. The domain, or what the input can be, is [0, ∞) as it can be greater than or equal to 0

The range is the total cost, or what the output can be. Since the total cost is the number of tickets multiplied by 55 (as it is $55 per ticket), this can be represented by 55 * number of tickets. The total cost cannot be negative, but it can be 0 if no one buys tickets. As there is no limit to the amount of tickets someone can buy, there is no limit to what the total cost could be, making the range [0, ∞)

the regular price of base ball cleats is 80 dollars if the cleats are on sale for 45% off, then what is the value of the discount in dollars?

Answers

Answer:

$36

Step-by-step explanation:

If the cleats were on sale for 45% off, that means they were 55% "on." Therefore, you have to multiply 80 by 0.55* to get how much they cost.

80 x 0.55 is 44.

To get the value of the discount, you subtract 44 from 80, which is 36.

The cleats sold for $44, and there was a $36 discount.

*you multiply by 0.55 because it is 55 percent. Percent means "per every hundred." That's why you have to divide 55 by 100 to get 0.55.

An alternate way to solve this is to multiply 80 by 55 and then divide the result by 100, but you'll still get 44.

Angle A and angle B are supplementary. If the measure of angle B is one-third the measure of angle A, what is the measure of angle B?
E. 22.5°
F. 45°
G. 67.5°
H. 135°

Answers

Answer:

vertically opposite angles

Step-by-step explanation:

supplementaries angles

The measure of angle B is 45° since option F. is correct in the given question.

What do we mean by supplementary angles?

Any two angles (say X & Y), are said to be supplementary when their summation (X + Y) is equal to 180°.

How to determine Angle B?

Angle A and Angle B are supplementary, which implies A+B=180°.

Angle B is one-third angle A, which implies

[tex]B=\frac{1}{3} A[/tex]

or, 3B = A

Using the value of A in the previous equation, we get 3B + B = 180°.

4B = 180°

or, B = 45°.

Thus, the measure of angle B is 45°. The correct answer is option F.

Learn more about supplementary angles on

https://brainly.com/question/404323

#SPJ2

What are the coordinates of R?

Answers

Answer: y,0

Step-by-step explanation:

What is the constant of variation, k, of the direct variation, y = for, through (5,8)?

Answers

Answer:

[tex]\frac{8}{5}[/tex]

Step-by-step explanation:

The constant of direction variation givens a proportion that is maintained by both x and y values for all points of a line that it passes through.

Usually represented with the variable [tex]k[/tex], it is given by:

[tex]\frac{y}{x}=k[/tex] for coordinates (x, y).

This relationship can be written as [tex]y=kx[/tex] which is also the layout for a proportional relationship.

Since coordinates are written (x, y), for point (5, 8), substitute [tex]x=5, y=8[/tex] to get the constant of variation:

[tex]8=5k,\\k=\boxed{\frac{8}{5}}[/tex]

Answer:

8/5

Step-by-step explanation:

Given that y varies directly with x , therefore ,

[tex]\implies y \propto x[/tex]

Let k be the constant . Therefore ,

[tex]\implies y = k x[/tex]

When the point is (5,8) ,

[tex]\implies 8 = k * 5 \\\\\implies \underline{\underline{\boxed{ k =\dfrac{8}{5}}}}[/tex]

Hence the constant of variation is 8/5.

The elevation of a city is 2633 feet above sea level.
Write a signed number to represent this elevation

Answers

Answer:

+2633 ft

Step-by-step explanation:

The city is above sea level meaning it is a positive number.

I have a goal of getting exactly $100. But I have to pay a tax of 10%. How much money do I need to get in order to have exactly $100 after paying tax? Explain your answer please.

Answers

Answer:  $111.11

=======================================================

Explanation:

Let's say you earn x dollars.

You pay 10% tax on that, which means you pay 0.10x dollars.

This would drop to x-0.10x = 0.90x which is the amount earned after tax.

So you pay 10% of what you earn, and keep 90% of the rest.

Set this equal to the target $100 you want and solve for x.

0.90x = 100

x = 100/0.90

x = 111.11

Notice that 10% of this is 0.10*x = 0.10*111.11 = 1.11 when rounding to the nearest penny. So you'll pay $1.11 in tax. Therefore, you'd have x-0.10x = 111.11 - 1.11 = 100 left over. So the answer checks out.

I NEED HELP SOMEONE HELP ME OUTTTT!!!

Answers

Answer:

Tan A = [tex]\frac{21}{35}[/tex] = [tex]\frac{3}{5}[/tex] = .6

Step-by-step explanation:

SOH CAH TOA

Sin Ф = [tex]\frac{O}{H}[/tex]

Cos Ф = [tex]\frac{A}{H}[/tex]

Tan Ф = [tex]\frac{O}{A}[/tex]

A rectangle has an area of 32 square millimeters. The length of the rectangle is 888 millimeters. What is the width of the rectangle?

Answers

Answer:

0.036

Step-by-step explanation:

See image below:)

the area is 32 and the length is 888

you use the formula w= a/l

so a= 32 and l= 888

The width is 0.036mm.

What is area of a rectangle?

Area of a rectangle (A) is the product of its length (l) and width (w).

A= l. w

Here, given that,

the area is 32 and the length is 888

We use the formula w= A/l

so, A= 32 and l= 888

Then, width= w= 32/888

                         =0.036 mm.

Hence, width is 0.036mm

To learn more on Area click:

https://brainly.com/question/20693059

#SPJ2

Which fraction is the largest? 7/9 3/4 1/2 2/3

Answers

Rewrite the fractions as decimals by dividing:

7/8 = 0.7777

3/4 = 0.75

1/2 = 0.5

2/3 = 0.666

0.777 is the largest number so 7/9 is the largest fraction.

Answer: 7/9

Answer:

2/3

Step-by-step explanation:

L.C.M:36

7/9:28/36

3/4:27/36

1/2:1836

2/3:36/36

How many numbers? How many numbers are there from 421 to 430? How many numbers are there from 567 to 573? How many numbers are there from 890 to 898?

what method you used to find out the numbers between one number and another

Answers

Answer:

9,6,8 but if you mean like all the numbers its infinite because you can add decimals and the number will never end

use the quadratic formula to solve x^2 + 3^x - 28 = 0

Answers

Step-by-step explanation:

x^2 + 3x - 28 = 0

x^2 - 7x + 4x - 28 = 0

x(x-7) + 4(x-7) = 0

(x+4)(x-7)= 0

x+4=0 or x-7=0

x= -4 or x=7

a_(n)=7n-1 9 ( what is the fourth term)

PLEASE HELP ASAP.

Answers

Answer:

Fourth term of given arithmetic progression a(4) = 30

Step-by-step explanation:

Given incomplete information:

a(n) = 7n - 1 9

Assume Correct information:

a(n) = 7(n - 1) + 9

Find:

Fourth term of given arithmetic progression

Computation:

Fourth term of given arithmetic progression = a(4)

So,

Number of term (n) = 4

So,

a(n) = 7(n - 1) + 9

a(4) = 7(4 - 1) + 9

a(4) = 7(3) + 9

a(4) = 21 + 9

a(4) = 30

Fourth term of given arithmetic progression a(4) = 30

which statement best describes a line in slope-intercept form when the coefficient of the x-trm is negative

Answers

Answer:

it's a negative slope

Step-by-step explanation:

**btw; not all slopes with a negative cefficient will look exactly like this, but as log as it has a negative coefficient, it will be negative and look somewhat similar**

What is the equation of the following line? Be sure to scroll down first to see
all answer options
(0,0)
2,-8)
O A. y=-4x
O B. y = 8x
O C. y=-x
D. y = 8
O E. y = 4x
O F. y = 18

Answers

Answer:

A

Step-by-step explanation:

The equation of a line in slope- intercept form is

y = mx + c ( m is the slope and c the y- intercept )

Calculate m using the slope formula

m = [tex]\frac{y_{2}-y_{1} }{x_{2}-x_{1} }[/tex]

with (x₁, y₁ ) = (0, 0) and (x₂, y₂ ) = (2, - 8) ← 2 points on the line

m = [tex]\frac{-8-0}{2-0}[/tex] = [tex]\frac{-8}{2}[/tex] = - 4

The line crosses the y- axis at (0, 0) ⇒ c = 0 , then

y = - 4x + c , that is

y = - 4x → A

The equation of line passes through the points (0, 0) and (2, -8) will be;

⇒ y = - 4x

What is Equation of line?

The equation of line in point-slope form passing through the points

(x₁ , y₁) and (x₂, y₂) with slope m is defined as;

⇒ y - y₁ = m (x - x₁)

Where, m = (y₂ - y₁) / (x₂ - x₁)

Given that;

Two points on the line are (0, 0) and (2, -8).

Now,

Since, The equation of line passes through the points (0, 0) and (2, -8).

So, We need to find the slope of the line.

Hence, Slope of the line is,

m = (y₂ - y₁) / (x₂ - x₁)

m = (- 8 - 0) / (2 - 0)

m = - 8 / 2

m = - 4

Thus, The equation of line with slope - 4 is,

⇒ y - 0 = - 4 (x - 0)

⇒ y = - 4x

Therefore, The equation of line passes through the points  (0, 0) and

(2, -8) will be;

⇒ y = - 4x

Learn more about the equation of line visit:

https://brainly.com/question/18831322

#SPJ7

PLEASE HELP PLEASE!!!!! NO LINKS!!!

Answers

Answer:

Step-by-step explanation:

a)   profits = 30,000 + (30,000 * .05 * years)

P(y) = 30,000 + 1500y

b)  P(15) =  30,000 + 1500*(15)

P(15) = 52,500

Ayudenme con esto porfavoor

Answers

Figura 1 = ⅓

Figura 2 = ½

Figura 3 = 4/7

Figura 4 = 4/9

Answer:[tex]\huge{ \frac{1}{3}} [/tex][tex]\huge{ \frac{1}{2}} [/tex][tex]\huge{ \frac{4}{7}}[/tex][tex]\huge{ \frac{4}{9}} [/tex]

↦ Count all the shaded regions (for numerator) & all the parts of the figure (for denominator).

ʰᵒᵖᵉ ⁱᵗ ʰᵉˡᵖˢ

꧁❣ ʀᴀɪɴʙᴏᴡˢᵃˡᵗ2²2² ࿐

If px-3+qx+1=2x+3,Find the value of p+q

Answers

Answer: p + q = (2x+5)/x

isolate p + q on one side: therefore rewrite the equation.

px + qx -2 = 2x + 3

px + qx = 2x + 5

x(p+q) = 2x+5

p + q = (2x+5)/x

What conclusion would be true given the hypothesis?

Answers

Answer:

4th option

Step-by-step explanation:

If ∠ 1 = ∠ 2 = ∠ 3

Then ∠ 1 = ∠ 3 is true

Answer:

The second one the measure of angle?1 is less then angle 3 is the correct answer.

help me please i need this right now !!

Answers

Answer:

The first choice.

Step-by-step explanation:

3(x + 2) is equal to 3 * x + 3 * 2.

Simplify that, and you would get 3x + 6.

Answer:

A. or 3(x+2) = 3x + 6

Step-by-step explanation:

A. 3(x+2) = 3(x) + 3(2)

               = 3x + 6

Which is correct!.

B. x^2 is nowhere to be found.

C. Doesn't distribute the 3 into the 2 properly.

D.  Doesn't distribute the -3 into the x properly.

Show all work to solve 3x2 - 5x – 2 = 0.

Answers

Answer:

[tex]3x\cdot \:2-5x-2=0[/tex]

[tex]6x-5x-2=0[/tex]

[tex]6x-5x=x[/tex]

[tex]x-2=0[/tex]

[tex]x-2+2=0+2[/tex]

[tex]x=2[/tex]

[tex]-----------[/tex]

Hope it helps..

Have a great day!!

Answer:

2, -1/3

Step-by-step explanation:

3x^2 - 5x - 2 = 0

Find the factors of 3*-2 that are the same as the addends for -5:

3*-2 = -6

Factors of -6: -6*1

Addends: -6 + 1

3x^2 - 6x + x - 2 = 0

(3x^2 - 6x) + (x - 2) = 0

(factor in each parantheses)

3x(x - 2) + 1(x - 2) = 0

(3x + 1)(x - 2) = 0

(set each of the terms to 0 and solve)

3x + 1 = 0

3x = -1

x = -1/3

x - 2 = 0

x = 2

Hope it helps (●'◡'●)

If U={a,b,c,d,e,f,g,h,} A={a,b,c,d} B={a,e,h}. Then verify the relation AUB

Answers

Answer:

U={a,b,c,d,e,f,g,h}

A={a,b,c,d}

now,

AUB= {a,b,c,d,e,f,g,h} U {a,b,c,d}

= {a,b,c,d,e,

ok,this question is wrong btw ,I think I stead of U(universal set) it is B

There is a confusion with the font used here as U has two different meanings and needs two different symbols.

The universal set is {a, b, c, d, e, f, g, h}

AUB is the union of set A and set B, that is all the elements in either A or B
= {a, b, c, d, e, h}
f and g are not in the union as they are not part of either A or B

If 12 + 6n is 20 percent bigger than k, what is k? 12 + 6n A. 5 B. 10 + 5n C. 2 +n 6(12 +6n) D. 5 E. 20 + 10n​

Answers

Answer:

k*1.2=6n+12

k=5(n+2)

2x+6=x-4
Solve this equation

Answers

The solution and answer are well written in the Pic above.

Answer:

x = 2

Step-by-step explanation:

2x  + 6 = x -4

Subtract 6 from both sides

2x  = x - 4 +6

2x = x + 2

Subtract 'x' from both sides

2x - x = 2

x = 2

Other Questions
what is 2x + 4 = x + 40 could i get an explanation on how to do this? The radius of a circle is 6 cm. What is its diameter? Which of these powers is granted to the federal government alone?levy a sales tax on state purchasesdeclare war or make treatiestry crimes that occur in a municipality levy taxes on imported goods A palindrome is a number that is identical when written both forwards and backwards. For example, 181, 25852, and 3333 are all palindromes. How many palindromes between 100 and 1000 are divisible by 3? How has texture proved essential to many cultural art forms?a.It has allow cultures to provide realistic imagery to future generations.b.It has been used to depict a message or a feeling.c.It provides mood and foreshadowing that would otherwise be lost.d.It has led to the creation of new art mediums. Find the intersection of the parabola y=x^2+4x+3 and the line x-y=-1 What could pioneers do in jumping-off towns? Check all that apply.buy wagonsbuy suppliesenjoy the quiet environmenthire guides to accompany themhave equipment and tools repaired which statement reflects a citizens need to increase his or her civic participation Mrs. Rodger got a weekly raise of $145. If she gets paid every other week, write an integer describing how the raise will affect her paycheck. At what x value does the function given below have a hole?f(x)=x+3/x29 The exclusionary rule does NOT apply in: Group of answer choices Habeas corpus proceedings. The exclusionary rule does not apply in A, B, or C. Parole revocation hearings. A how are proteins and nucleic acids related When a respondent is offered explicit responses from which to choose an answer to a question, they are answering a(n) HELP PLEASE I WILL GIVE BRAINLIEST FOR THE RIGHT ANSWERS After completing the group discussion, what did you learn about the text? Did other students analyze the story differently than you? If so, how? Which ideas were supported with reasons and evidence from the text, and which ideas were not? PLZ HELP MEEEEE For each journey between work and home, Arjun uses 1.3 gallons of petrol. Arjun has 52 litres of petrol in his car. How many complete journeys between work and home can he do?1 gallon = 4.5 litres. PLZZ HELP I REALLY DONT WANNA GO DOWN!!!!!!!!!!!! Please solve the question attached: a concrete has a height of 5m and has unit area 3m supports a mass of 30000kg. Determine the stress, strain and change in height